Wednesday, March 30, 2016

Question 3 -- 2016, Inventory Control



Question 3: Assume a single item deterministic situation with constant demand and lead time.
a)      Derive a formula for finding Economic Order Quantity (EOQ).                     2 marks
b)      For an order quantity Q1, the total ordering cost is X and inventory carrying cost is Y. X is less than Y. If we change the order quantity to Q2, total ordering cost X gets reduced. Which is closer to EOQ; Q1 or Q2? Why?                                                     3 marks



 Answer video can be viewed and downloaded from https://drive.google.com/file/d/0B6PHuk6I5SxbODhITFhfMnM3M3M/view?usp=sharing.


1 comment:

  1. Common mistakes and marking pattern:

    a) Some only wrote the formula. 0 marks for that. Some took the two cost components equal and solved the equation to get the formula. They got 0 or 1 depending on the explanation given. Those who derived it properly but didn’t check for max or min by differentiating again too got full marks here.

    b) A valid point related to cost, order quantity and EOQ has got 1 mark. Few students have done it right either by drawing the graph and getting Q1 and Q2 both on right side of EOQ or by arguing on the difference between the two cost components. They have got full marks. Rest 0.

    ReplyDelete